Anatomy

Pataasin ang iyong marka sa homework at exams ngayon gamit ang Quizwiz!

81 A 16-year-old boy is admitted to the hospital with fever, a confused mental state, and drowsiness. During physical examination it is noted that the boy suffers from severe acne. Radiologic examination reveals cavernous sinus thrombosis. Which of the following routes of entry to the cavernous sinus would most likely be responsible for the infection and thrombosis? A. Carotid artery B. Mastoid emissary vein C. Middle meningeal artery D. Ophthalmic vein E. Parietal emissary vein

81 D. Cavernous sinus thrombosis can often result from squeezing pimples or other infectious processes located around the danger area of the face, which includes the area of the face directly surrounding the nose. This physical pressure has the potential to move infectious agents from the pimple into the ophthalmic vein, which then carries it to the cavernous sinus. The pterygoid venous plexus and ophthalmic vein both communicate with the cavernous sinus and therefore offer a route of travel for the spread of infection, but the path provided by the superior ophthalmic vein is a more direct route. Additionally, the superior ophthalmic vein receives blood supply from the supraorbital, supratrochlear, and angular veins that supply the area around the nose and lower forehead. (Venous blood in the head can flow in either direction because these veins do not possess valves.) The emissary veins communicate between the venous sinuses and the veins of the scalp and would therefore not be involved in the spread of infection between the nose and cavernous sinus. The middle meningeal artery courses between the dura and periosteum, whereas the carotid artery, specifically the ICA, traverses through the cavernous sinus and provides origin to the ophthalmic artery. As with the middle meningeal artery, the carotid artery would not offer a route of communication between the area of infection and the cavernous sinus

190 A 55-year-old male farmer is admitted to the emergency department after falling from the hayloft in his barn. Radiographic examination reveals a small, depressed fracture of the skull vertex and thrombosis of the superior sagittal sinus. A day later the patient loses consciousness. What is the most likely cause of his loss of consciousness? A. Obstruction of CSF resorption B. Obstruction of the cerebral aqueduct (of Sylvius) C. Laceration of the middle meningeal artery D. Fracture of the cribriform plate with CSF rhinorrhea E. Aneurysm of the middle cerebral artery

A. CSF is mostly secreted from the choroid plexuses of the lateral, third, and fourth ventricles of the brain. The CSF enters the subarachnoid space from the fourth ventricle, via the foramina of Luschka and Magendie. The CSF then circulates in the subarachnoid space until it is finally resorbed back into the venous side of the circulation through the arachnoid granulations into the superior sagittal sinus. A thrombus of the superior sagittal sinus can to lead to an obstruction of CSF (communicating hydrocephalus) in which all of the ventricles of the brain are enlarged and the intracranial pressure is increased. GAS 31-32; N 109-111; McM 69-71

155 After a fall on her outstretched arm, a 72-year-old woman presents with elbow pain. Physical examination reveals a palpable defect over her biceps brachii tendon. Elbow flexion causes pain but does not limit active movement. Radiographs do not show fractures or dislocations. She is diagnosed with a biceps brachii tendon rupture. Which of the following muscles most likely allow the patient to continue to flex her elbow? A. Brachialis and brachioradialis B. Flexor carpi ulnaris and flexor carpi radialis C. Flexor digitorum superficialis and flexor digitorum profundus D. Pronator teres and supinator E. Triceps brachii and coracobrachialis

A. Flexion of the elbow is achieved by contraction of the biceps brachii, brachialis, and brachioradialis muscles. The brachialis muscle is the major flexor of the elbow joint and together with the brachioradialis will continue to achieve flexion if the biceps brachii is damaged. The flexor carpi ulnaris and radialis produce flexion of the wrist, and the flexor digitorum superficialis and profundus produce flexion of the digits at the metacarpophalangeal and interphalangeal joints, respectively. The pronator teres and supinator are responsible for pronation and supination, respectively. The coracobrachialis does not cross the elbow joint and acts only on the shoulder, while the triceps brachii is the elbow extensor

127 A 43-year-old victim of a drunk driving car crash is undergoing reconstructive arm surgery. The surgeon performs an autograft using a weak adductor of the leg located superficially on the medial side of the thigh. Which muscle is most likely being harvested to perform this reconstruction? A. Gracilis B. Sartorius C. Rectus femoris D. Vastus lateralis E. Vastus medialis

A. Gracilis due to its shape, size, and more importantly the nature of neurovascular supply is used very commonly in reconstructive surgery as a free functioning autograft.

107 A 34-year-old man is lifting heavy weights while doing squats. Unfortunately, while making a maximal effort, he drops the weight and immediately grabs at his upper thigh, writhing in pain. The man is admitted to the emergency department and during physical examination is diagnosed with a femoral hernia. What reference structure would be found immediately lateral to the herniated structures? A. Femoral vein B. Femoral artery C. Pectineus muscle D. Femoral nerve E. Adductor longus muscle

A. In a femoral hernia, abdominal contents are forced through the femoral ring, which is just lateral to the lacunar ligament (of Gimbernat) and just medial to the femoral vein. The femoral vein would be found immediately lateral to the femoral hernia. This is correct in most cases because in the majority of people, the femoral vein is found more medial to both the femoral artery and nerve in the femoral triangle. The adductor longus muscle as well as the pectineus muscle would be found deep and medial to the hernia.

30 In an accident during cleanup of an old residential area of the city, the Achilles tendon of a 32-year-old worker was cut through by the blade of a brush cutter. The patient is admitted to the hospital and a laceration of the Achilles tendon is diagnosed. Which of the following bones serves as an insertion for the Achilles tendon? A. Calcaneus B. Fibula C. Cuboid D. Talus E. Navicular

A. The Achilles tendon inserts upon the calcaneus bone. This tendon represents a combination of the tendons of gastrocnemius and soleus muscles. The tendon of the plantaris can insert with this tendon.

53 A 58-year-old female dancer presented to the orthopedic clinic with a complaint of pain during her work because of bilateral bunions. She was referred to a podiatric surgeon who scheduled her for surgery. The protruding bony and soft tissues of the toe were excised, and a muscle was reflected from the lateral side of the proximal phalanx, together with a sesamoid bone, upon which the muscle also inserted. What muscle was this? A. Adductor hallucis B. Abductor hallucis C. First dorsal interosseous D. First lumbrical E. Quadratus plantae

A. The adductor hallucis muscle inserts upon the lateral side of the proximal phalanx of the great toe, and also the lateral sesamoid bone, by way of its oblique and transverse heads. It is supplied by the lateral plantar nerve. The abductor hallucis inserts upon the medial side of the proximal phalanx and the medial sesamoid bone of the great toe. The sesamoid bones are within the tendon of the flexor hallucis brevis and assist it in its function at the first metatarsophalangeal joint. The abductor and flexor hallucis brevis are innervated by the medial plantar nerve. The first dorsal interosseous muscle and the first lumbrical both insert on the medial side of the extensor mechanism of the second toe. The quadratus plantae arises from the calcaneus and inserts on the tendon of the flexor digitorum longus muscle. The first lumbrical is supplied by the medial plantar nerve. The quadratus plantae, the lumbricals 2 to 4, and all interossei are innervated by the lateral plantar nerve.

9 A 39-year-old man presents with severe neck pain after a whiplash injury, sustained when his car was struck from behind. Radiologic studies reveal trauma to the ligament lying on the anterior surface of the cervical vertebral bodies. Which ligament is most likely disrupted? A. Anterior longitudinal ligament B. Ligamentum flavum C. Nuchal ligament D. Posterior longitudinal ligament E. Transverse cervical ligament

A. The anterior longitudinal ligament lies anterior to the vertebral bodies along the vertebral column. The ligamentum flavum connects the laminae of two adjacent vertebrae. The nuchal ligament is a continuation of the supraspinous ligament above C7, which connects spinous processes. The posterior longitudinal ligament lies on the posterior margin of the vertebral bodies. The transverse cervical (cardinal) ligament is associated with the pelvic region of the body and not the spinal column

39 The kidneys of a 32-year-old woman were failing, and she needed to be placed on dialysis. However, the search in her upper limb for a suitable vein was unexpectedly difficult. The major vein on the lateral side of the arm was too small; others were too delicate. Finally, a vein was found on the medial side of the arm that passed through the superficial and deep fascia to join veins beside the brachial artery. Which of the following veins was this? A. Basilic B. Lateral cubital C. Cephalic D. Medial cubital E. Medial antebrachial

A. The basilic vein can be used for dialysis, especially when the cephalic vein is judged to be too small, as in this case. The basilic vein can be elevated from its position as it passes through the fascia on the medial side of the arm (brachium). The cephalic vein passes more laterally up the limb. The lateral cubital vein is a tributary to the cephalic vein, and the medial cubital vein joins the basilic vein, both of which are rather superficial in position. The medial antebrachial vein courses up the midline of the forearm (antebrachium) ventrally

112 A 29-year-old man is brought to the physician for removal of a cast from his left leg. He had sustained a fracture of the left lower extremity 6 weeks prior which was immobilized in a cast that extended from just below the knee to the foot. At the time of injury, there was severe pain but normal strength in the extremity. When the cast was removed, physical examination showed a pronounced left foot drop with paresthesia and sensory loss over the dorsum of the left foot and lateral leg. Injury to which nerve is the most likely cause? A. Common fibular (peroneal) B. Superficial fibular (peroneal) C. Deep fibular (peroneal) D. Sciatic E. Tibial

A. The common fibular (peroneal) nerve is a branch of the sciatic nerve. It descends on the lateral side of the popliteal fossa before winding around the head of the fibula. It then divides into superficial and deep nerves that supply the lateral and anterior compartments of the leg respectively. Due to its superficial course, it is easily injured in patients with long leg casts (which run from just below the knee). The nerve supplies the dorsiflexors of the leg, the skin of the first web space (via the deep fibular), the evertors of the foot, and the skin of the lateral side of the leg and dorsum of the foot (via the superficial fibular).

176 A 25-year-old man falls on a slippery trail and injures his elbow and hand. Inspection reveals abrasions over the olecranon, medial epicondyle, and palm of the hand. Physical examination reveals decreased sensation with "pins and needles" (paraesthesia) along the ulnar border of the hand and medial one and a half digits. There is also weakness of finger abduction/ adduction, thumb adduction, and flexion at the DIP of the ring and little fingers. Which structure was most likely injured? A. Ulnar nerve at the medial epicondyle B. Ulnar nerve at Guyon's canal C. Median nerve in the cubital fossa D. Median nerve in the carpal tunnel E. Medial cord of brachial plexus in the axillary inlet

A. The deficits describe ulnar nerve damage close to its entry into the forearm. The ulnar nerve passes behind the medial epicondyle and is relatively unprotected, making this area prone to nerve injury. In the forearm, via its muscular branches, it innervates the flexor carpi ulnaris muscle and the medial half of the flexor digitorum profundus muscle. In the hand the deep branch of the ulnar nerve innervates the hypothenar muscles, adductor pollicis, abductor digiti minimi, flexor digiti minimi brevis, third and fourth lumbricals, opponens digiti minimi, and palmaris brevis muscles. The sensory innervation is to the fifth and medial half of the fourth digit and corresponding part of the hand, which can explain the deficits experienced by the patient.

102 A 7-year-old boy is undergoing a surgery to remove a tumor from his spinal cord. During surgery of the spinal cord, which of the following structures is used as a landmark to identify anterior rootlets from posterior rootlets? A. Denticulate ligament B. Filum terminale C. Conus medullaris D. Posterior longitudinal ligament E. Ligamenta flava

A. The denticulate ligament is a sheet of pia mater running longitudinally on either side of the spinal cord, connecting it to the dura mater. Medially, the denticulate ligament lies between the origin of the anterior and posterior rootlets serving as a landmark to differentiate between them. The conus medullaris is the terminal end of the spinal cord and the filum terminale is an extension of the pia mater that connects the conus medullaris to the dural sac. The posterior longitudinal ligament lies posterior to the vertebral bodies, while the ligamentum flavum connects the lamina of adjacent vertebrae.

5 A 49-year-old male construction worker is admitted to the emergency department with a painful lump on the proximal medial aspect of his thigh. Radiologic and physical examinations reveal that the patient has a herniation of abdominal viscera beneath the inguinal ligament into the thigh. Through which of the following openings will a hernia of this type initially pass to extend from the abdomen into the thigh? A. Femoral ring B. Superficial inguinal ring C. Deep inguinal ring D. Fossa ovalis E. Obturator canal

A. The femoral ring is the abdominal opening of the femoral canal. A femoral hernia passes through the femoral ring into the femoral canal deep and inferior to the inguinal ligament. It can appear as a bulging at the saphenous hiatus (fossa ovalis) of the deep fascia of the thigh, the hiatus through which the saphenous vein passes to the femoral vein. The superficial inguinal ring is the triangular opening in the aponeurosis of the external abdominal oblique and lies lateral to the pubic tubercle. The deep inguinal ring lies in the transversalis fascia lateral to the inferior epigastric vessels. Herniation into either of these two openings is associated with an inguinal hernia. The obturator canal, a bony opening between the superior and inferior ramus of the pubic bone, is the site of an obturator hernia.

34 A 21-year-old female softball pitcher is examined in the emergency department after she was struck in the arm by a line drive. Plain radiographic and MRI studies show soft tissue injury to the region of the spiral groove, with trauma to the radial nerve. Which of the following muscles would be intact after this injury? A. Flexor carpi ulnaris B. Extensor indicis C. Brachioradialis D. Extensor carpi radialis longus E. Supinator

A. The flexor carpi ulnaris muscle is not innervated by the radial nerve but rather by the ulnar nerve. The brachioradialis, extensor carpi radialis longus and brevis, and supinator muscles are all innervated by the radial nerve distal to the spiral groove

14 A 45-year-old man is admitted to the hospital after accidentally walking through a plate glass door in a bar while intoxicated. Physical examination shows multiple lacerations to the upper limb, with inability to flex the distal interphalangeal joints of the fourth and fifth digits. Which of the following muscles is most likely affected? A. Flexor digitorum profundus B. Flexor digitorum superficialis C. Lumbricals D. Flexor digitorum profundus and flexor digitorum superficialis E. Interossei

A. The flexor digitorum profundus muscle is dually innervated by the ulnar nerve to the medial two fingers and the median nerve for the long and index fingers. Because of the superficial course of the ulnar nerve, it is vulnerable to laceration. Such an injury would result in an inability to flex the distal interphalangeal joints of the fourth and fifth digits because the flexor digitorum profundus muscle is the only muscle that flexes this joint. The flexor digitorum superficialis muscle is innervated by the median nerve only, and the course of this nerve runs too deep to be usually affected by lacerations. The lumbricals function to flex the MP joints and assist in extending the IP joints. The interossei adduct and abduct the fingers.

147 A 21-year-old painter sustains a laceration on the anterior surface of his left wrist just distal to the skin fold crease. When he arrives at the emergency department, the physician extends the patient's wrist to determine the depth of the laceration and observes a broad, glistening white structure deep to the superficial fascia. The patient has no numbness or tingling of any of the fingers and is able to discriminate sharp/dull sensation in all of the fingers and palm of the hand. There is no loss of motion in any of the fingers or the hand, and grip strength is normal. Which structure is the physician most likely observing? A. Flexor retinaculum B. Flexor carpi ulnaris tendon C. Palmar skin D. Flexor digitorum superficialis tendons E. Flexor digitorum profundus tendons

A. The flexor retinaculum is a thick connective tissue ligament that spans the space between the medial and lateral sides of the base of the carpal tunnel. It protects and stabilizes the tendons that run beneath it. Damage to the flexor carpi ulnaris tendon, flexor digitorum superficialis tendons, and flexor digitorum profundus tendons result in functional losses in the hand. The palmar skin is loose connective tissue and does not have a shiny, glistening appearance

89 A 67-year-old woman had a bad fall while walking her dog the evening before. She states that she fell on her outstretched hand. Radiographs do not demonstrate any bony fractures. The clinician observes the following signs of neurologic injury: weakness of flexion of her wrist in a medial direction, a loss of sensation on the medial side of the hand, and clawing of the fingers. Where is the most likely place of nerve trauma? A. Behind the medial epicondyle B. Between the pisiform bone and the flexor retinaculum C. Within the carpal tunnel D. At the cubital fossa, between the ulnar and radial heads of origin of flexor digitorum superficialis E. At the radial neck, 1 cm distal to the humerocapitellar joint

A. The force of the woman's fall on the outstretched hand was transmitted up through the forearm, in this case resulting in dislocation of the olecranon at the elbow, putting traction on the ulnar nerve as it passes around the medial epicondyle of the humerus. Ulnar trauma at the elbow can cause weakness in medial flexion (adduction) at the wrist, from loss of the flexor carpi ulnaris. Ulnar nerve injury also results in sensory loss in the medial hand and paralysis of the interossei and medial two lumbricals, with clawing especially of digits 4 and 5. Injury of the ulnar nerve at the pisiform bone would not affect the flexor carpi ulnaris, nor would it produce sensory loss on the dorsum of the hand because the dorsal cutaneous branch of the ulnar branches off proximal to the wrist. Carpal tunnel problems affect median nerve function, which is not indicated here. The ulnar nerve passes medial to the cubital fossa between the heads of the flexor carpi ulnaris, not between the heads of the flexor digitorum superficialis. Injuries at the radial neck affect the site of division of the radial nerve, and its paralysis would not result in the clinical problems seen in this patient

17. A 32-year-old patient received a badly placed intramuscular injection to the posterior part of his gluteal region. The needle injured a motor nerve in the area. Later, he had great difficulty rising to a standing position from a seated position. Which muscle was most likely affected by the injury? A. Gluteus maximus B. Gluteus minimus C. Hamstrings D. Iliopsoas E. Obturator internus

A. The gluteus maximus is innervated by the inferior gluteal nerve, and this muscle is responsible for extension and lateral rotation of the thigh. It is the primary muscle that extends the flexed hip and is used to rise from a seated position. The gluteus minimus is innervated by the superior gluteal nerve and is responsible for abduction of the thigh. Hamstring muscles are innervated by the tibial portion of the sciatic nerve, and these are responsible for extension of the thigh and flexion of the leg. The iliopsoas muscle is innervated by L1 and L2 and the femoral nerve, and flexes the thigh. The obturator internus is innervated by the nerve to the obturator internus and is a lateral rotator of the thigh.

114 A 55-year-old man is admitted to the emergency department after slipping on wet pavement and falling. Physical examination reveals that the patient has a hematoma that formed in the danger zone of the scalp, spreading to the area of the eyelids. Which of the following layers is regarded as the "danger zone"? A. Loose, areolar layer B. Skin C. Galea aponeurotica D. Pericranium E. Subcutaneous layer

A. The loose areolar connective tissue layer is known as the "danger zone" because hematoma can spread easily from this layer into the skull by means of emissary veins that pass into and through the bones of the skull.

50 A 22-year-old male construction worker is admitted to the hospital after he suffers a penetrating injury to his upper limb from a nail gun. Upon physical examination, the patient is unable to flex the distal interphalangeal joints of digits 4 and 5. What is the most likely cause of his injury? A. Trauma to the ulnar nerve near the trochlea B. Trauma to the ulnar nerve at the wrist C. Median nerve damage proximal to the pronator teres D. Median nerve damage at the wrist E. Trauma to spinal nerve root C8

A. The nail was fired explosively from the nail gun and then pierced the ulnar nerve near the coronoid process of the ulna and the trochlea of the humerus. Paralysis of the medial half of the flexor digitorum profundus muscle would result (among other significant deficits), with loss of flexion of the distal interphalangeal joints of digits 4 and 5. Ulnar trauma at the wrist would not affect the interphalangeal joints, although it would cause paralysis of interossei, hypothenar muscles, and so on. Median nerve damage proximal to the pronator teres would affect proximal interphalangeal joint flexion and distal interphalangeal joint flexion of digits 2 and 3 as well as thumb flexion. Median nerve injury at the wrist would cause loss of thenar muscles but not long flexors of the fingers. Trauma to spinal nerve ventral ramus C8 would affect all long finger flexors

111 A 29-year-old woman is involved in a car crash and is taken to the emergency department. Radiographs reveal a fracture of her pelvis. During healing of the pelvic fracture, a nerve becomes entrapped in the bone callus. Musculoskeletal examination reveals an inability to adduct the thigh. Which of the following nerves is most likely affected? A. Obturator B. Femoral C. Inferior gluteal D. Superior gluteal E. Tibial

A. The obturator nerve is a branch of the lumbar plexus that originates from L2 to L4. It descends medial to the psoas on the posterior abdominal wall into the pelvis where it runs along the lateral wall of the lesser pelvis, above and anterior to the obturator vessels. It enters into the medial thigh via the obturator canal (an opening above the obturator membrane) to supply the obturator externus muscle and the adductors of the thigh. The femoral nerve innervates the anterior compartment of the thigh. The inferior gluteal innervates the gluteus maximus muscle, while the superior gluteal innervates the gluteus minimus and medius. The tibial nerve innervates the posterior compartment of the lower limb.

70 A 12-year-old boy lacerated the palmar surface of the wrist while playing with a sharp knife. The cut ends of a tendon could be seen within the wound in the exact midline of the wrist. Which tendon lies in this position in most people? A. Palmaris longus B. Flexor carpi radialis C. Abductor pollicis longus D. Flexor carpi ulnaris E. Flexor pollicis longus

A. The palmaris longus passes along the midline of the flexor surface of the forearm. The flexor carpi radialis is seen in the lateral portion of the forearm superficially, passing over the trapezium to insert at the base of the second metacarpal. The abductor pollicis longus tendon is laterally located in the wrist, where it helps form the lateral border of the anatomic snuffbox. The flexor carpi ulnaris tendon can be seen and palpated on the medial side of the wrist ventrally. The flexor pollicis longus tendon passes deep through the carpal tunnel.

66 The patellar reflex appears to be markedly reduced in a 33-year-old diabetic female patient, due to deficient vascular supply of the nerves of her lower limb. The tendon of which of the following muscles is stretched during the patellar reflex? A. Quadriceps femoris B. Quadratus femoris C. Sartorius D. Pectineus E. Biceps femoris

A. The patellar ligament is a very heavy ligament that connects the patella to the tibial tuberosity; it provides the insertion of the quadriceps femoris tendon upon the tibia. The patella can be thought of as a bone (a sesamoid bone) that develops within the tendon of the quadriceps femoris muscle. When the reflex hammer strikes the patellar ligament, it stretches the ligament slightly for a brief time, resulting in reflex contraction of the quadriceps femoris muscles. This reflex arc is elicited by the femoral nerve (L4 sensory input component and L2, L3 motor output). The quadriceps femoris includes the rectus femoris and the vastus lateralis, intermedius, and medialis. The patella is the largest sesamoid bone in the body. A sesamoid bone is a bone that develops within a tendon. The quadratus femoris muscle of the gluteal area arises from the ischial tuberosity and inserts on the femur proximally. The sartorius arises from the anterior superior iliac spine and inserts on the proximal, medial aspect of the tibia as one of the three tendinous components of the pes anserinus (goose foot). The biceps femoris of the posterior thigh has a long head that arises from the ischial tuberosity and a short head that arises from the femur; they insert on the head of the fibula.

56 A 61-year-old man was hit in the midhumeral region of his left arm by a cricket bat. Physical examination reveals an inability to extend the wrist and loss of sensation on a small area of skin on the dorsum of the hand proximal to the first two fingers. What nerve supplies this specific region of the hand? A. Radial B. Posterior interosseous C. Lateral antebrachial cutaneous D. Medial antebrachial cutaneous E. Dorsal cutaneous of ulnar

A. The patient has suffered injury to the radial nerve in the midhumeral region. The nerve that provides sensation to the dorsum of the hand proximal to the thumb and index finger is the superficial branch of the radial nerve. The posterior interosseous nerve supplies a strip of skin on the back of the forearm and wrist extensors. The lateral antebrachial cutaneous nerve is a continuation of the musculocutaneous nerve and supplies the lateral side of the forearm. The medial antebrachial cutaneous is a direct branch of the medial cord and supplies skin of the medial side of the forearm. The dorsal cutaneous branch of the ulnar nerve supplies the medial side of the dorsum of the hand

20 A 25-year-old male athlete is admitted to the emergency department after a bad landing while performing the pole vault. Radiographic examination of his hand reveals a fractured carpal bone in the floor of the anatomic snuffbox (Fig. 6-2). Which bone has most likely been fractured? A. Triquetral B. Scaphoid C. Capitate D. Hamate E. Trapezoid

B. The anatomic snuffbox is formed by the tendons of the extensor pollicis brevis, the abductor pollicis longus, and the extensor pollicis longus. The floor is formed by the scaphoid bone, and it is here that one can palpate for a possible fractured scaphoid.

118 A 34-year-old woman is admitted to the emergency department after a car crash. Radiographic studies show marked edema and hematoma of the arm, but there are no fractures. During physical examination the patient presents with inability to abduct her arm without first establishing lateral momentum of the limb, and inability to flex the elbow and shoulder. Which of the following portions of the brachial plexus is most likely injured? A. Superior trunk B. Middle trunk C. Inferior trunk D. Lateral cord E. Medial cord

A. The superior trunk of the brachial plexus includes C5 and C6, which give rise to the suprascapular nerve, which innervates the supraspinatus muscle. The supraspinatus muscle is the primary muscle involved in abduction of the arm from 0 to 15 degrees. The deltoid muscle, supplied primarily by C5, abducts the arm from 15 to 90 degrees. The middle trunk is just C7 and has nothing to do with the muscle involved in initial abduction of the arm. The inferior trunk is C8-T1 and does not supply the supraspinatus muscle; therefore, it is not the right answer. The cords are distal to the branching of the supraspinatus muscle; therefore, neither lateral cord nor medial cord is the correct answer.

96. An 85-year-old man is admitted to the hospital with a painful arm after lifting a case of wine. Physical examination gives evidence of a rupture of the long tendon of the biceps brachii (Fig. 6-6). Which of the following is the most likely location of the rupture? A. Intertubercular groove B. Midportion of the biceps brachii muscle C. Junction with the short head of the biceps brachii muscle D. Proximal end of the combined biceps brachii muscle E. Bony insertion of the muscle

A. The tendon of the long head of the biceps brachii muscles runs in the intertubercular groove on the proximal humerus as it changes direction and turns medially to attach to the supraglenoid tubercle of the scapula. This change in direction within an osseous structure predisposes the tendon to wear and tear, particularly in people who overuse the biceps brachii muscle. This type of injury presents with a characteristic sign called the "Popeye sign" after the cartoon character

97 A 37-year-old woman had been suffering for months from piriformis entrapment syndrome, which was not relieved by physical therapy. Part of the sciatic nerve passed through the piriformis, and a decision was made for surgical resection of the muscle. When the area of entrapment was identified and cleared, a tendon could be seen emerging through the lesser sciatic foramen, at first hidden by two smaller muscles and several nerves and vessels destined for the region of the perineum. The tendons of which of the following muscles pass through this opening? A. Obturator internus B. Obturator externus C. Quadratus femoris D. Gluteus minimus E. Gluteus medius

A. The tendon of the obturator internus leaves the pelvic cavity by passing through the lesser sciatic foramen, wrapping around the lesser sciatic notch, changing direction by about 90 degrees. It is joined there by the superior and inferior gemelli and inserts with them on the upper portion of the greater trochanter. The obturator externus arises on the external surface of the pubic bone and obturator membrane and inserts on the greater trochanter. The quadratus femoris arises from the ischial tuberosity and inserts on the intertrochanteric line of the femur. The gluteus medius and minimus insert together on the lateral aspect of the greater trochanter

124 A 34-year-old man is admitted to the hospital after a car collision. Radiographic examination reveals a fracture at his wrist. Physical examination reveals paralysis of the muscles that act to extend the interphalangeal joints (Fig. 6-9). Which of the following nerves is most likely injured? A. Ulnar B. Recurrent branch of median C. Radial D. Musculocutaneous E. Anterior interosseous

A. The ulnar nerve innervates the dorsal and palmar interossei, which act to abduct and adduct the fingers and assist the lumbricals in their actions of flexing the metacarpophalangeal joints and extending the interphalangeal joints. The recurrent branch of the median nerve innervates the thenar muscle group that functions in the movement of the thumb. The radial and musculocutaneous nerves do not innervate any muscles in the hand. The anterior interosseous innervates the flexor pollicis longus and the pronator quadratus.

6 An 18-year-old man is brought to the emergency department after an injury while playing rugby. Imaging reveals a transverse fracture of the humerus about 1 inch proximal to the epicondyles. Which nerve is most frequently injured by the jagged edges of the broken bone at this location? A. Axillary B. Median C. Musculocutaneous D. Radial E. Ulnar

B. A supracondylar fracture often results in injury to the median nerve. The course of the median nerve is anterolateral, and at the elbow it lies medial to the brachial artery on the brachialis muscle. The axillary nerve passes posteriorly through the quadrangular space, accompanied by the posterior circumflex humeral artery, and winds around the surgical neck of the humerus. Injury to the surgical neck may damage the axillary nerve. The musculocutaneous nerve pierces the coracobrachialis muscle and descends between the biceps brachii and brachialis muscle. It continues into the forearm as the lateral antebrachial cutaneous nerve. The ulnar nerve descends behind the medial epicondyle in its groove and is easily injured and produces "funny bone" symptoms

58 A 22-year-old male football player suffered a wrist injury while falling with force on his outstretched hand. When the anatomic snuffbox is exposed in surgery, which artery is visualized crossing the fractured bone that provides a floor for this space? A. Ulnar B. Radial C. Anterior interosseous D. Posterior interosseous E. Deep palmar arch

B. As the radial artery passes from the ventral surface of the wrist to the dorsum, it crosses through the anatomic snuffbox, passing over the scaphoid bone. The ulnar artery at the wrist is located on the medial side of the wrist, passing from beneath the flexor carpi ulnaris to reach Guyon's canal between the pisiform bone and the flexor retinaculum. Guyon's canal is adjacent to but not in communication with the carpal tunnel. The anterior interosseous and posterior interosseous arteries arise from the common interosseous branch of the ulnar artery and pass proximal to distal in the forearm between the radius and ulna, in the flexor and extensor compartments, respectively. The deep palmar branch of the ulnar artery passes between the two heads of the adductor pollicis to anastomose with the radial artery in the palm

52 The arachnoid villi allow cerebrospinal fluid to pass between which two of the following spaces? A. Choroid plexus and subdural space B. Subarachnoid space and superior sagittal sinus C. Subdural space and cavernous sinus D. Superior sagittal sinus and jugular vein E. Epidural and subdural space

B. The arachnoid villi are extensions of the arachnoid mater into the superior sagittal sinus. The villi allow for proper drainage of the CSF into the venous bloodstream from the subarachnoid space in which the CSF circulates. These villi are a crucial element in maintaining proper intracranial pressure and circulation of the CSF

101 A 32-year-old man is admitted to the emergency department with visual problems. Radiographic examination reveals a tumor of the adenohypophysis (anterior pituitary gland). Physical examination reveals a loss of the lateral halves of the fields of vision of both eyes (bitemporal hemianopia or "tunnel vision"). Which of the following structures was most likely compressed by the tumor? A. Optic nerve B. Optic chiasm C. Optic tract D. Oculomotor E. Abducens nerve

B. Compression of the optic chiasm can cause bitemporal hemianopia due to compression of nerve fibers coming from the nasal (medial) hemiretinas of both eyes. The optic chiasm is located in very close proximity above the pituitary gland. Compression of an optic nerve would cause complete blindness in the affected eye. Compression of an optic tract would cause homonymous hemianopia. Compression of the oculomotor nerve would cause the eye to deviate "out and down" (paralysis of the four extraocular muscles innervated by this nerve), ptosis (paralysis of levator palpebrae), and mydriasis (paralysis of constrictor pupillae). Compression of the abducens nerve would cause paralysis of the lateral rectus muscle, leading to medial deviation (adduction) of the eye

43 A 49-year-old male worker fell from a ladder, with his weight impacting on the heels of his feet. Radiologic examination reveals comminuted calcaneal fractures. After the injury the contraction of which one of the following muscles could most likely increase the pain in the injured foot? A. Flexor digitorum profundus B. Gastrocnemius C. Tibialis posterior D. Tibialis anterior E. Fibularis (peroneus) longus

B. Contraction of the gastrocnemius on the fractured calcaneus would increase the pain because the gastrocnemius inserts with the soleus upon that bone, via the calcaneal tendon, or tendo Achilles. The flexor digitorum profundus passes the ankle medially to enter the sole of the foot, where it inserts upon the distal phalanges. The tibialis posterior, likewise, passes under the medial malleolus, with complex insertions upon the navicular bone, cuneiform bones, metatarsal bones, and the cuboid bone. The tibialis anterior, a muscle of the anterior leg compartment, inserts upon the navicular bone and, with the tibialis posterior, is a strong invertor of the foot. The fibularis (peroneus) longus is a muscle of the lateral compartment of the leg. It passes under the lateral malleolus, entering the sole of the foot by crossing the lateral surface of the calcaneus, and inserts primarily into the medial cuneiform and base of the first metatarsal bone

101 A 22-year-old pregnant woman was admitted urgently to the hospital after her baby had begun to appear at the introitus. The baby had presented in the breech position, and it had been necessary to exert considerable traction to complete the delivery. The newborn is shown in Fig. 6-7. Which of the following structures was most likely injured by the trauma of childbirth? A. Radial nerve B. Upper trunk of the brachial plexus C. Lower trunk of the brachial plexus D. Median, ulnar, and radial nerves E. Upper and lower trunks of the brachial plexus

B. During a breech delivery as described here, downward traction is applied to the shoulders and upper limbs as the baby is forcibly extracted from the birth canal. This exerts traction on the upper cord of the brachial plexus, often causing a traction injury from which the baby can often recover. If the ventral rami of C5 and C6 are avulsed from the spinal cord, the injury is permanent.

64. A 43-year-old woman visits the outpatient clinic with a neurologic problem. Diagnostically, she cannot hold a piece of paper between her thumb and the lateral side of her index finger without flexing the distal joint of her thumb. This is a positive Froment's sign, which is consistent with ulnar neuropathy. Weakness of which specific muscle causes this sign to appear? A. Flexor pollicis longus B. Adductor pollicis C. Flexor digiti minimi D. Flexor carpi radialis E. Extensor indicis

B. Froment's sign is positive for ulnar nerve palsy. More specifically it tests the action of the adductor pollicis muscle. The patient is asked to hold a sheet of paper between the thumb and a flat palm. The flexor pollicis longus is innervated by the anterior interosseous branch of the median nerve. The flexor digiti minimi is innervated by the deep branch of the ulnar nerve and would not be used to hold a sheet of paper between the thumb and palm. The flexor carpi radialis is innervated by the median nerve, and the extensor indicis is innervated by the radial nerve

5 A 64-year-old man arrived at the clinic with a painful rash and skin eruptions that are localized entirely on one side of his body, closely following the dermatome level of the spinal nerve C7. The patient was diagnosed with the a herpes zoster virus infection known as "shingles." In what structure has the virus most likely proliferated to cause the patient's current condition? A. The sympathetic chain B. The dorsal root ganglion of the C7 spinal nerve C. The lateral horn of the C7 spinal cord segment D. The posterior cutaneous branch of the dorsal primary ramus of C7 E. The ventral horn of the C7 spinal cord segment

B. Herpes zoster is a viral disease that remains latent in the dorsal root ganglia of the sensory nerves and when the virus becomes active presents as a painful skin lesion. It is associated only with sensory nerve fibers and has no motor involvement. The only answer choice that is solely responsible for sensory innervation is the dorsal root ganglion.

3 A 30-year-old man suffered a superior gluteal nerve injury in a motorcycle crash in which his right lower limb was caught beneath the bike. He is stabilized in the emergency department. Later he is examined and he exhibits a waddling gait and a positive Trendelenburg sign. Which of the following would be the most likely physical finding in this patient? A. Difficulty in standing from a sitting position B. The left side of the pelvis droops or sags when he attempts to stand with his weight supported just by the right lower limb C. The right side of the pelvis droops or sags when he attempts to stand with his weight supported just by the left lower limb D. Weakened flexion of the right hip E. Difficulty in sitting from a standing position

B. Injury to the superior gluteal nerve results in a characteristic motor loss, with paralysis of the gluteus medius and minimus. In addition to their role in abducting the thigh, the gluteus medius and minimus function to stabilize the pelvis. When the patient is asked to stand on the limb of the injured side, the pelvis descends on the opposite side, indicating a positive Trendelenburg test. The gluteal, or lurching, gait that results from this injury is characterized by the pelvis drooping to the unaffected side when the opposite leg is raised. In stepping forward, the affected individual leans over the injured side when lifting the good limb off the ground. The uninjured limb is then swung forward. The gluteus maximus, supplied by the inferior gluteal nerve, is the main muscle responsible for allowing a person to rise to a standing position (extending the flexed hip). Spinal nerve roots L1 and L2 and the femoral nerve are responsible for hip flexion. Injury to the left superior gluteal nerve would result in sagging of the right side of the pelvis when the affected individual stands on the left limb. The hamstring muscles, mainly responsible for flexing the knees to allow a person to sit down from a standing position, are innervated by the tibial branch of the sciatic nerve.

16 Upon removal of a knee-high leg cast, a 15-yearold boy complains of numbness of the dorsum of his right foot and inability to dorsiflex and evert his foot. Which is the most probable site of the nerve compression that resulted in these symptoms? A. Popliteal fossa B. Neck of the fibula C. Lateral compartment of the leg D. Anterior compartment of the leg E. Medial malleolus

B. The common fibular (peroneal) nerve winds around the neck of the fibula before dividing into superficial and deep branches that go on to innervate the lateral and anterior compartments of the leg, respectively. These compartments are responsible for dorsiflexion and eversion of the foot, and injury to these nerves would result in deficits in these movements. The tibial nerve lies superficially in the popliteal fossa. This nerve innervates the posterior compartment of the leg, so compression in this area would result in a loss of plantar flexion and weakness of inversion. The lateral compartment of the leg is innervated by the superficial fibular (peroneal) nerve and is mainly involved in eversion of the foot. The cutaneous branches of the superficial fibular (peroneal) nerve emerge through the deep fascia in the anterolateral aspect of the leg and supply the dorsum of the foot. The anterior compartment of the leg is innervated by the deep fibular (peroneal) nerve and is mainly involved in dorsiflexion of the foot. The medial malleolus is an inferiorly directed projection from the medial side of the distal end of the tibia. The tibial nerve runs near the groove behind the medial malleolus, and compression at this location would result in loss of toe flexion, adduction, abduction, and abduction of the great toe.

36 A 34-year-old man complains of hyperacusis (sensitivity to loud sounds). Injury to which of the following cranial nerves is most likely responsible? A. Hypoglossal B. Facial C. Accessory D. Vagus E. Glossopharyngeal

B. The facial nerve innervates the stapedius muscle, which is responsible for limiting movement of the stapes, thereby reducing the intensity of the sound entering the inner ear. The hypoglossal nerve innervates tongue muscles; the spinal accessory nerve supplies the trapezius and sternocleidomastoid muscles; the vagus nerve does not provide any innervation for sound in the ear; and the glossopharyngeal nerve only supplies sensation to the posterior third of the tongue, pharynx (gag reflex), middle ear cavity, and tympanic membrane, and muscle innervation to the stylopharyngeus muscle. GAS 895, 898-899; N 96; McM 60, 61

105 A 49-year-old man is admitted to the emergency department with a cold and pale foot. Physical examination reveals that the patient suffers from peripheral vascular disease; his popliteal artery is occluded and no pulse is felt upon palpation. What is the landmark to feel the pulse of the femoral artery? A. Adductor canal B. Femoral triangle C. Popliteal fossa D. Inguinal canal E. Pubic symphysi

B. The femoral triangle is the best place to palpate the femoral pulse. It is bounded by the sartorius muscle laterally, adductor longus medially, and the inguinal ligament superiorly. It contains the femoral vein, artery, and nerve (from medial to lateral, respectively). The adductor canal lies deep between the anterior and medial compartments of the thigh and therefore cannot be palpated. The popliteal fossa is the fossa at the back of the knee and contains the popliteal artery and vein, tibial nerve, and common fibular (peroneal) nerve. The femoral pulse cannot be palpated here

116 During an interview, a 30-year-old man who is a psychiatric patient suddenly becomes aggressive. In order to calm him down, the patient is given an intramuscular injection in the upper lateral quadrant of the buttock. The injection is given at this specific location to prevent damage to which of the following nerves? A. Lateral femoral cutaneous B. Sciatic C. Superior gluteal D. Obturator E. Inferior gluteal

B. The gluteal region can be divided into quadrants by two lines positioned using palpable bony landmarks. One line runs inferiorly from the highest point of the iliac crest. The second line runs horizontally midway between the iliac crests and the ischial tuberosity. This divides the gluteal region into four quadrants. The sciatic nerve runs through the lower medial quadrant and must be avoided during intragluteal injections.

96 In preparing to isolate the proximal portion of the femoral artery, the vascular surgeon gently separated it from surrounding tissues. Posterior to the femoral sheath, what muscle forms the lateral portion of the floor of the femoral triangle? A. Adductor longus B. Iliopsoas C. Sartorius D. Pectineus E. Rectus femoris

B. The iliopsoas forms the lateral portion of the troughlike floor of the femoral triangle. The pectineus forms the medial portion of this floor. The adductor longus provides a medial border for the femoral triangle and meets the sartorius, the lateral border of the triangle, at the apex. The rectus femoris is a superficial contributor to the quadriceps femoris, lateral to the femoral triangle

89 A 34-year-old woman has a direct blow to the patella by the dashboard of the vehicle during an automobile crash. The woman is admitted to the emergency department and radiologic examination reveals patellofemoral syndrome. This type of syndrome is characterized by lateral dislocation of the patella. Which of the following muscles requires strengthening by physical rehabilitation to prevent future dislocation of the patella? A. Vastus lateralis B. Vastus medialis C. Vastus intermedius D. Rectus femoris E. Patellar ligament

B. The lower portion of the vastus medialis inserts upon the medial aspect of the patella and draws it medially, especially in the last quarter of extension—during which it is especially palpable in contraction. This lower portion of the muscle is referred to as the vastus medialis obliquus (VMO). Increasing the strength of this muscle can lessen the lateral dislocation of the patella. The rectus femoris arises from the anterior inferior iliac spine and lip of the acetabulum and draws the patella vertically upward, as does the vastus intermedius.

114 A 59-year-old woman is admitted to the hospital in a state of shock. During physical examination, several lacerations are noted in her forearm and her radial pulse is absent. Where is the most typical place to identify the radial artery immediately after crossing the radiocarpal joint? A. Between the two heads of the first dorsal interosseous muscle B. In the anatomic snuffbox C. Below the tendon of the flexor pollicis longus D. Between the first and second dorsal interossei muscles E. Between the first dorsal interosseous muscle and the adductor pollicis longus

B. The radial artery enters the palm through to pierce through the two heads of the first dorsal interosseous muscle and enter the deep aspect of the palm. The flexor pollicis longus tendon runs on the palmar aspect of the hand and the radial artery runs on the dorsal aspect of the hand before entering the deep aspect of the palm, and therefore the radial artery does not run below this tendon. The radial artery does not run between the first and second interosseous muscle and therefore cannot be used as a landmark to identify the artery. Finally, the artery does not run between the first dorsal interosseous muscle and the adductor pollicis longus.

8 A 32-year-old woman is admitted to the emergency department after an automobile collision. Radiologic examination reveals multiple fractures of the humerus. Flexion and supination of the forearm are severely weakened. She also has loss of sensation on the lateral surface of the forearm. Which of the following nerves has most likely been injured? A. Radial B. Musculocutaneous C. Median D. Lateral cord of brachial plexus E. Lateral cutaneous nerve of the forearm

B. The musculocutaneous nerve supplies the biceps brachii and brachialis muscles, which are the flexors of the forearm at the elbow. The musculocutaneous nerve continues as the lateral antebrachial cutaneous nerve, which supplies sensation to the lateral side of the forearm (with the forearm in the anatomic position). The biceps brachii muscle is the most powerful supinator muscle. Injury to this nerve would result in weakness of supination and forearm flexion and lateral forearm sensory loss. Injury to the radial nerve would result in weakened extension and a characteristic wrist drop. Injury to the median nerve causes paralysis of flexor digitorum superficialis muscle and other flexors in the forearm and results in a characteristic flattening of the thenar eminence. The lateral cord of the brachial plexus gives origin both to the musculocutaneous and lateral pectoral nerves. There is no indication of pectoral paralysis or weakness. Injury to the lateral cord can result in weakened flexion and supination in the forearm, and weakened adduction and medial rotation of the arm. The lateral cutaneous nerve of the forearm is a branch of the musculocutaneous nerve and does not supply any motor innervation. Injury to the musculocutaneous nerve alone is unusual but can follow penetrating injuries.

15 A 56-year-old man with advanced bladder carcinoma suffers from difficulty while walking. Muscle testing reveals weakened adductors of the right thigh. Which nerve is most likely being compressed by the tumor to result in walking difficulty? A. Femoral B. Obturator C. Common fibular (peroneal) D. Tibial E. Sciatic

B. The obturator nerve arises from the lumbar plexus and enters the thigh through the obturator canal. This nerve is responsible for innervation of the medial compartment of the thigh (adductor compartment). Injury to this nerve can result in weakened adduction and difficulty walking. The femoral nerve innervates muscles of the anterior compartment of the thigh that are responsible for hip flexion and leg extension. The sciatic nerve branches into the common fibular (peroneal) and tibial nerves. The common fibular (peroneal) nerve branches into the deep and superficial branches of the fibular (peroneal) nerve responsible for innervation of the anterior and lateral compartments of the leg, respectively. The tibial nerve innervates the muscles of the posterior compartment of the thigh and leg, which are responsible for extension of the hip, flexion of the leg, and plantar flexion of the foot

59 A 45-year-old intoxicated man was struck by a tour bus while walking in the middle of the street. The man was admitted to the emergency department and during physical examination was diagnosed with "adductor gait," in which an individual crosses one limb in front of the other, due to powerful hip adduction. Which of the following nerves was most likely involved in this condition? A. Tibial B. Obturator C. Inferior gluteal D. Superior gluteal E. Femoral

B. The obturator nerve innervates the adductor muscles, including the gracilis, pectineus, and obturator externus. The tibial nerve supplies the calf muscles and intrinsic muscles in the plantar portion of the foot. The inferior gluteal nerve innervates the gluteus maximus; the superior gluteal nerve supplies the gluteus medius and minimus and tensor fasciae latae. The femoral nerve provides motor supply to the quadriceps femoris, sartorius, and, in some cases, the pectineus. This gait pattern is characteristic of hypertonia in the lower limb. As a result these areas become flexed to various degrees, giving the appearance of crouching, while tight adductors produce extreme adduction.

117 A 52-year-old man is admitted to the emergency department after falling on wet pavement. Radiographic examination reveals fracture of the radius. An MRI study reveals a hematoma between the fractured radius and supinator muscle. Upon physical examination the patient has weakened abduction of the thumb and extension of the metacarpophalangeal joints of the fingers. Which of the following nerves is most likely affected? A. Anterior interosseous B. Posterior interosseous C. Radial nerve D. Deep branch of ulnar nerve E. Median nerve

B. The posterior interosseous nerve is an extension of the deep branch of the radial nerve after it emerges distal to the supinator. It is responsible for innervation of several muscles in the extensor compartment of the posterior aspect of the forearm, including extension of the metacarpophalangeal joints. The deep radial nerve courses laterally around the radius and passes between the two heads of the supinator muscle and is thus likely to be compressed by a hematoma between the fractured radius and the supinator muscle. Though the radial nerve gives rise to the posterior interosseous nerve, this answer choice is too general and would not indicate the precise injured branch of the radial nerve. Both the deep branch of the ulnar nerve and the median nerve traverse the medial and anteromedial aspect of the arm, respectively. These nerves primarily supply the flexor compartment of the arm. The anterior interosseous nerve is a branch of the median nerve and supplies the flexor digitorum profundus, flexor pollicis longus, and the pronator quadratus muscles

17 A 19-year-old man is diagnosed with a herniated disc but he has no symptoms of spinal cord injury. In the event of intervertebral disc herniation in the cervical region, which of the following ligaments is in an anatomic position to protect the spinal cord from direct compression? A. Supraspinous B. Posterior longitudinal C. Anterior longitudinal D. Ligamentum flavum E. Nuchal ligament

B. The posterior longitudinal ligament is the only ligament spanning the posterior aspect of the vertebral bodies and intervertebral discs. With intervertebral disc herniation, the nucleus pulposus of the intervertebral disc protrudes posterolaterally. The anterior longitudinal ligament traverses the anterior side of the vertebral bodies and thus would not protect the spinal cord from direct compression. The supraspinous and ligamentum flavum ligaments connect the spinous processes and the laminae of adjacent vertebrae, respectively. The nuchal ligament is a continuation of the supraspinous ligaments near the C7 vertebrae and runs to the occipital protuberance.

152 A 53-year-old woman reports difficulty in walking. Physical examination showed a positive Trendelenburg sign when she is asked to stand on her right leg. Which nerve has been compromised to produce the positive sign? A. Sciatic B. Right superior gluteal C. Left inferior gluteal D. Left superior gluteal E. Right inferior gluteal

B. The right superior gluteal nerve is the correct choice. When a person stands on one leg or walks, the gluteus medius, gluteus minimus, and tensor muscles of the fascia latae act in synergy to stabilize the hip joint by abducting the hip (pelvic tilt). These muscles receive their innervation from the superior gluteal nerve. The abductors of the hip, as they contract to maintain the stability of the hip joint, draw the pelvis forcefully toward the weigh-bearing leg, causing the opposite side of the pelvis to tilt in that same direction. The right superior gluteal nerve innervates its ipsilateral medius, minimus, and tensor muscles of the fascia latae. Loss of these muscles results in a positive Trendelenburg sign with the pelvis dropping on the left side

93 A 27-year-old male triathlon competitor complained that he frequently experienced deep pains in one calf that almost caused him to drop out of a regional track-and-field event. Doppler ultrasound studies indicated, and surgical exposure confirmed, the existence of an accessory portion of the medial head of the gastrocnemius that was constricting the popliteal artery. Above the medial head of the gastrocnemius, the superior medial border of the popliteal fossa could be seen. Which of the following structures forms this border? A. Tendon of biceps femoris B. Tendons of semitendinosus and semimembranosus C. Tendon of plantaris D. Adductor hiatus E. Popliteus

B. The tendons of the semitendinosus and semimembranosus provide the superior medial border of the popliteal fossa. The semitendinosus inserts with the pes anserinus on the proximal, medial tibia. The semimembranosus inserts on the tibia posteriorly. The biceps femoris forms the superior lateral border of the fossa, as the tendon passes to insertion on the fibula. The plantaris arises from the femur just above the lateral head of the gastrocnemius, passing distally to insert on the calcaneus via the tendo Achilles. The popliteus arises from the tibia and passes superiorly and laterally to insert on the lateral condyle of the femur, with a connection to the lateral meniscus.

104 A 20-year-old man visits the family physician complaining of difficulty to flex and medially rotate his thigh while running and climbing. Which of the following muscles is most likely damaged in this individual? A. Rectus femoris B. Tensor fasciae latae C. Vastus intermedius D. Semimembranosus E. Sartorius

B. The tensor fasciae latae assists in flexion of the thigh, as well as medial rotation and abduction. Damage to this muscle would adversely affect these motions. The rectus femoris extends the hip. The vastus intermedius extends the knee. The semimembranosus extends the hip and flexes and medially rotates the knee. The sartorius assists in flexion and lateral rotation of the hip, as well as in medial rotation of the knee

50 A 22-year-old man is admitted to the emergency department after falling from his bicycle. Radiologic examination reveals a fracture of the tibia above the ankle. MRI and physical examination reveal that the tibial nerve is severed on the posterior aspect of the tibia. Which of the following signs will most likely be present during physical examination? A. Sensory loss of the dorsum of the foot B. Sensory loss on the sole of the foot C. Foot drop D. Paralysis of the extensor digitorum brevis E. Sensory loss of the entire foot

B. The tibial nerve divides into the medial and lateral plantar nerves on the medial side of the ankle. These two nerves provide sensation for the sole of the foot. Sensory supply to the dorsum of the foot is provided mostly by the superficial fibular (peroneal) nerve, with the deep fibular (peroneal) nerve providing sensation for the skin between the first and second toes. Foot drop would be caused by interruption of the common fibular (peroneal) nerve. Sensory loss to the lateral side of the foot results from loss of the sural nerve. Paralysis of the extensor digitorum brevis would be attributed to injury to the terminal motor branch of the deep fibular (peroneal) nerve.

5 While walking to his classroom building, a firstyear medical student slipped on the wet pavement and fell against the curb, injuring his right arm. Radiographic images showed a midshaft fracture of the humerus. Which pair of structures was most likely injured at the fracture site? A. Median nerve and brachial artery B. Axillary nerve and posterior circumflex humeral artery C. Radial nerve and deep brachial artery D. Suprascapular nerve and artery E. Long thoracic nerve and lateral thoracic artery

C. A midshaft humeral fracture can result in injury to the radial nerve and deep brachial artery because they lie in the spiral groove located in the midshaft. Injury to the median nerve and brachial artery can be caused by a supracondylar fracture that occurs by falling on an outstretched hand and partially flexed elbow. A fracture of the surgical neck of the humerus can injure the axillary nerve and posterior circumflex humeral artery. The suprascapular artery and nerve can be injured in a shoulder dislocation. The long thoracic nerve and lateral thoracic artery may be damaged during a mastectomy procedure

183 A 58-year-old man is admitted to the ENT clinic with progressive unilateral hearing loss and ringing in the affected ear (tinnitus) of 4 months duration. MRI reveals a tumor at the cerebellopontine angle. Which of the following nerves is most likely affected? A. Vagus B. Hypoglossal C. Vestibulocochlear D. Glossopharyngeal E. Trigeminal

C. A tumor at the cerebellopontine angle, such as an acoustic schwannoma, is most likely to affect first the vestibulocochlear nerve and then the facial nerve. This excludes the vagus, hypoglossal, glossopharyngeal, and trigeminal nerves from being the correct answers.

161 A 48-year-old woman is seen in the orthopedic clinic with symptoms of carpal tunnel syndrome. This could result in weakening of which muscles? A. Dorsal and palmar interossei B. Lumbricals III and IV C. Thenar and lumbricals I and II D. Flexor digitorum superficialis and profundus E. Hypothenar

C. Carpal tunnel syndrome is a relatively common condition that causes pain, numbness, and a tingling sensation in the hand and fingers. Carpal tunnel syndrome is caused by compression of the median nerve, which supplies the thenar muscles and the first and second lumbricals. Dorsal and palmar interossei and the hypothenar muscles are supplied by the ulnar nerve. The flexor muscles of the forearm are supplied by the median nerve before it passes through the carpal tunnel.

123 A 61-year-old man was hit by a cricket bat in the midhumeral region of his left arm. Physical examination reveals normal elbow motion; however, he could not extend his wrist or his metacarpophalangeal joints and he reported a loss of sensation on a small area of skin on the dorsum of the hand proximal to the first two digits. Radiographic examination reveals a hairline fracture of the shaft of the humerus just distal to its midpoint. Which of the following nerves is most likely injured? A. Median B. Ulnar C. Radial D. Musculocutaneous E. Axillary

C. Injury to the radial nerve can be caused by a blow to the midhumeral region because the nerve winds around the shaft of the humerus. The symptoms described include the loss of wrist and finger extension and a loss of sensation in an area of skin supplied by the radial nerve

21 A 42-year-old woman is diagnosed with stenosis of the cervical vertebral canal. A laminectomy of two vertebrae is performed. Which of the following ligaments will most likely also be removed? A. Anterior longitudinal B. Denticulate C. Ligamentum flavum D. Nuchal E. Cruciate

C. The anterior longitudinal ligament runs along the anterior-most aspect of the vertebral column from C1 to the sacrum and would therefore be unaffected by a laminectomy. Denticulate ligaments extend laterally from the pia mater to the arachnoid mater along the length of the spinal cord. The ligamentum flavum is one of the two ligaments found in the vertebral canal and is adherent to the anterior aspect of the vertebral arches and often greatly thickened in spinal pathology. It is thus simultaneously removed upon excision of the lamina. The nuchal ligament is a thick longitudinal extension continuing from the supraspinous ligament at the level of C7 to the external occipital protuberance (inion). The cruciate ligament is an incorrect answer because it is located anterior to the spinal cord, and thus would not be involved in laminectomy

44 The arterial circle (of Willis) contributes greatly to cerebral arterial circulation when one primary artery becomes occluded by atherosclerotic disease. Which of the following vessels does not contribute to the circle? A. Anterior communicating artery B. Posterior communicating artery C. Middle cerebral artery D. Internal carotid artery E. Posterior cerebral artery

C. The arterial circle (of Willis) receives its blood supply from the internal carotid and vertebral arteries. The actual circle is formed by the bifurcation of the basilar, posterior cerebral, posterior communicating, internal carotid, anterior cerebral, and anterior communicating arteries. The middle cerebral artery is the lateral continuation of the internal carotid artery and therefore not part of the arterial circle. Although it receives its blood supply from the arterial circle (of Willis), it does not actually form any part of the circle.

19 A 35-year-old man has a small but painful tumor under the nail of his little finger. Which of the following nerves would have to be anesthetized for a painless removal of the tumor? A. Superficial radial B. Common palmar digital of median C. Common palmar digital of ulnar D. Deep radial E. Recurrent branch of median

C. The common palmar digital branch comes off the superficial branch of the ulnar nerve and supplies the skin of the little finger and the medial side of the ring finger. The superficial branch of the radial nerve provides cutaneous innervation to the radial (lateral) dorsum of the hand and the radial two and a half digits over the proximal phalanx. The common palmar digital branch of the median nerve innervates most of the lateral aspect of the palmar hand and the dorsal aspect of the second and third finger as well as the lateral part of the fourth digit. The deep radial nerve supplies the extensor carpi radialis brevis and supinator muscles and continues as the posterior interosseous nerve. The recurrent branch of the median nerve supplies the abductor pollicis brevis, flexor pollicis brevis, and opponens pollicis muscles.

146 A 22-year-old woman who is in training to become a phlebotomist is performing venipuncture on another student. She places the needle into the median cubital vein but is unable to withdraw blood. She quickly realizes that she passed the needle completely through the vein. Which of the following structures located deep to the median cubital vein has acted as a barrier and has prevented her from puncturing an artery? A. Flexor retinaculum B. Pronator teres muscle C. Bicipital aponeurosis D. Brachioradialis muscle E. Biceps brachii tendon

C. The median cubital vein is a superficial vein that lies on the biceps brachii aponeurosis. The biceps brachii aponeurosis, also known as lacertus fibrosus, is a flat sheet of connective tissue that fans out from the medial side of the biceps brachii tendon to blend with the deep fascia of the biceps brachii muscle. It reinforces the cubital fossa and protects the brachial artery, which runs beneath it

160 A 65-year-old man is admitted to the emergency department after an episode of a transient ischemic attack. Radiographic examination reveals an aneurysm in the region between the posterior cerebral artery and superior cerebellar artery. Which of the following nerves will most likely be compressed from the aneurysm? A. Trochlear B. Abducens C. Oculomotor D. Vagus E. Optic

C. The oculomotor nerve passes between the posterior cerebral artery (PCA) and the superior cerebellar artery near the junction of the midbrain and pons. The optic nerve arises near the arterial circle of Willis close to the internal carotid artery. Its location would thus prevent compression following an aneurysm at the PCA and superior cerebellar artery. Although the trochlear nerve could be compressed by the superior cerebellar artery, it would not likely be damaged by an aneurysm of the PCA. The abducens nerve is located in the pons, and the vagus is situated near the postolivary sulcus in the medulla. Neither of these nerves is likely to be compressed by the arteries mentioned here due to their more distal location.

110 During a battle, a 19-year-old soldier is shot in the lateral aspect of the right foot by a bullet that ricocheted off a building. The soldier is taken to a field hospital. A radiograph of the foot reveals that the base of the fifth metatarsal was completely obliterated. Which of the following muscles is most likely affected by this injury? A. Tibialis anterior B. Fibularis (peroneus) longus C. Gastrocnemius D. Fibularis (peroneus) brevis E. Extensor hallucis longus

D. The fibularis (peroneal) brevis muscle originates from the lateral lower two thirds of the shaft of the fibula and inserts on the tubercle at the base of the fifth metatarsal. Any injury to this area will affect this muscle. Patients will present with a weakness in the eversion of the foot. Fibularis (peroneus) longus, extensor hallucis longus and tibialis anterior all insert on the medial side of the foot and will not be affected in this patient. The gastrocnemius inserts via the Achilles tendon to the posterior surface of the calcaneus.

7 A 52-year-old female band director suffered problems in her right arm several days after strenuous field exercises for a major athletic tournament. Examination in the orthopedic clinic reveals wrist drop and weakness of grasp but normal extension of the elbow joint. There is no loss of sensation in the affected limb. Which nerve was most likely affected? A. Ulnar B. Anterior interosseous C. Posterior interosseous D. Median E. Superficial radial

C. The radial nerve descends posteriorly between the long and lateral heads of the triceps brachii muscle and passes inferolaterally on the back of the humerus between the medial and lateral heads of the triceps brachii muscle. It eventually enters the anterior compartment and descends to enter the cubital fossa, where it divides into superficial and deep branches. The deep branch of the radial nerve winds laterally around the radius and runs between the two heads of the supinator muscle and continues as the posterior interosseous nerve, innervating extensor muscles of the forearm. Because this injury does not result in loss of sensation over the skin of the upper limb, it is likely that the superficial branch of the radial nerve is not injured. If the radial nerve were injured very proximally, the woman would not be able to extend her elbow. The branches of the radial nerve to the triceps brachii muscle arise proximal to where the nerve runs in the spiral groove. The anterior interosseous nerve arises from the median nerve and supplies the flexor digitorum profundus, flexor pollicis longus, and pronator quadratus muscles, none of which seem to be injured in this example. Injury to the median nerve causes a characteristic flattening (atrophy) of the thenar eminence

35 A 48-year-old man complains of diplopia (double vision). On neurologic examination he is unable to adduct his left eye and lacks a corneal reflex on the left side. Where is the most likely location of the lesion resulting in the symptoms? A. Inferior orbital fissure B. Optic canal C. Superior orbital fissure D. Foramen rotundum E. Foramen ovale

C. The superior orbital fissure is the opening that allows the passage of the oculomotor nerve and the trochlear nerve; the lacrimal, frontal, and nasociliary branches of ophthalmic division of the trigeminal nerve; the abducens nerve; the superior and inferior divisions of the ophthalmic vein; and the sympathetic fibers from the cavernous plexus. The sensory and motor components of the corneal reflex are the ophthalmic division of the trigeminal nerve and the oculomotor nerve, whereas the eye impairment is due to a lesion to the oculomotor nerve, all of which are transmitted through the superior orbital fissure. The inferior orbital fissure contains the maxillary division of the trigeminal nerve, infraorbital vessels, and branches of the pterygopalatine ganglion. The optic canal contains the ophthalmic artery and optic nerve, in addition to sympathetic fibers. The foramen rotundum contains the maxillary nerve. The foramen ovale contains the lesser petrosal nerve, the mandibular division of the trigeminal nerve, the accessory middle meningeal artery, and the emissary veins. GAS 901, 934; N 85; McM 23, 25

22 A 28-year-old pregnant woman is admitted to the obstetrics department for delivery. In the final stages of labor, a caudal anesthetic is administered via the sacral hiatus. Into which of the following spaces in the sacral canal is the anesthetic placed? A. Vertebral canal B. Vertebral venous plexus C. Epidural space D. Subarachnoid space E. Subdural space

C. The vertebral canal is the longitudinal canal that extends through the vertebrae, containing the meninges, spinal cord, and associated ligaments. The internal vertebral venous plexus is the mostly valveless network of veins extending longitudinally along the vertebral canal. Neither of these answer choices describes a specific space. The spinal epidural space is found superficially to the dura mater. It is a fat-filled space extending from C1 to the sacrum. The subarachnoid space is a true space containing CSF. It is found within the CNS and extends to the level of S2. The subdural space is a potential space between the dura and the arachnoid mater. Normally, these two layers are fused due to the pressure of CSF in the subarachnoid space.

63 A 45-year-old woman is admitted to the outpatient clinic for shoulder pain. During physical examination, she presents with weakened shoulder movements. Radiologic examination reveals signs of quadrangular space syndrome, causing weakened shoulder movements. Which of the following nerves is most likely affected? A. Suprascapular B. Subscapular C. Axillary D. Radial E. Ulnar

C. The weakness in shoulder movement results from denervation of the teres minor and deltoid by the axillary nerve, which passes through the quadrangular space. Quadrangular space syndrome occurs when there is hypertrophy of the muscles that border the quadrangular space or fibrosis of portions of the muscles that are in contact with the nerve.

18 In spinal anesthesia, the needle is often inserted between the spinous processes of the L4 and L5 vertebrae to ensure that the spinal cord is not injured. This level is safe because in the adult the spinal cord usually terminates at the disc between which of the following vertebral levels? A. T11 and T12 B. T12 and L1 C. L1 and L2 D. L2 and L3 E. L3 and L4

C. This is the location of the conus medullaris, a tapered conical projection of the spinal cord at its inferior termination. Although the conus medullaris rests at the level of L1 and L2 in adults, it is often situated at L3 in newborns. The cauda equina and filum terminale extend beyond the conus medullaris

128 A 41-year-old woman is admitted to the hospital after a car crash. Radiographic examination reveals a transverse fracture of the radius proximal to the attachment of the pronator teres muscle. The proximal portion of the radius is deviated laterally. Which of the following muscles will most likely be responsible for this deviation? A. Pronator teres B. Pronator quadratus C. Brachialis D. Supinator E. Brachioradialis

D. The supinator muscle attaches to the radius proximally and when fractured would cause a lateral deviation. The pronator teres muscle originates on the medial epicondyle and coronoid process of the ulna and inserts onto the middle of the lateral side of the radius, pulling the radius medially below the fracture. The pronator quadratus muscle originates on the anterior surface of the distal ulna and inserts on the anterior surface of the distal radius, pulling the radius medially. The brachioradialis muscle originates on the lateral supracondylar ridge of the humerus and inserts at the base of the radial styloid process, far below the fracture. The brachialis muscle originates in the lower anterior surface of the humerus and inserts in the coronoid process and ulnar tuberosity, hence not causing an action on the radius

91 A 48-year-old woman is admitted to the hospital with severe abdominal pain. Several imaging methods reveal that the patient suffers from intestinal ischemia. An abdominopelvic catheterization is ordered for antegrade angiography. A femoral puncture is performed. What is the landmark for femoral artery puncture? A. Halfway between anterior superior iliac spine and pubic symphysis B. 4.5 cm lateral to the pubic tubercle C. Midpoint of the inguinal skin crease D. Medial aspect of femoral head E. Lateral to the fossa ovalis

D. Femoral artery puncture is one of the most common vascular procedures. The femoral artery can be localized often by simply feeling for the strongest point of the femoral pulse just inferior to the inguinal ligament. The femoral artery can be accessed with fluoroscopic assistance at the medial edge of the upper portion of the head of the femur. It is easily localized by Doppler ultrasound if the pulse is difficult to detect, such as in an obese patient. It is here that catheters are passed into the femoral artery for catheterization of abdominopelvic and thoracic structures and for antegrade angiography. It is also a site where arterial blood can be obtained for gas analysis. The midinguinal point, halfway between the anterior superior iliac spine and the pubic symphysis, can be either medial or lateral to the femoral artery and is not a dependable landmark. A needle inserted at the level of the inguinal crease, or inferior to the femoral head, can enter the femoral artery distal to the origin of the deep femoral artery, presenting more risk for accidental vascular injury. Four centimeters lateral to the pubic tubercle overlies the deep inguinal ring, with potential entry to spermatic cord, femoral vein, or artery. The fossa ovalis is the opening in the deep fascia of the thigh for the termination of the great saphenous vein in the femoral vein

4 A 45-year-old man arrived at the emergency department with injuries to his left elbow after he fell in a bicycle race. Plain radiographic and magnetic resonance imaging (MRI) examinations show a fracture of the medial epicondyle and an injured ulnar nerve. Which of the following muscles will most likely be paralyzed? A. Flexor digitorum superficialis B. Biceps brachii C. Brachioradialis D. Flexor carpi ulnaris E. Supinator

D. Fracture of the medial epicondyle often causes damage to the ulnar nerve due to its position in the groove behind the epicondyle. The ulnar nerve innervates one and a half muscles in the forearm, the flexor carpi ulnaris and the medial half of the flexor digitorum profundus muscles. The nerve continues on to innervate most of the muscles in the hand. The flexor digitorum superficialis is innervated by the median nerve and the biceps brachii muscle by the musculocutaneous. The radial nerve innervates both the brachioradialis and supinator muscles

178 A 54-year-old woman is admitted to the emergency department after a serious motor vehicle accident. Physical examination shows soft tissue edema and bruising around the neck. A radiograph of the humeroscapular region reveals a fracture of the midhumerus. Which of the following areas will most likely have impaired or absent sensation? A. Lateral aspect of the forearm B. Medial aspect of the arm C. Medial aspect of the arm and forearm D. Posterior aspect of the forearm E. Lateral and posterior aspect of the forearm

D. In the midshaft region of the humerus the radial nerve runs in the radial groove; fracture of the humerus at this point will likely impinge directly on the radial nerve, producing a sensory deficit along the posterior aspect of the forearm. The lateral aspect of the forearm is innervated by the lateral antebrachial cutaneous nerve of the forearm, which comes from the musculocutaneous nerve. These nerves may not be affected by a midshaft fracture of the humerus because they are well separated from the bone by muscle. The medial aspect of the arm and forearm is supplied by the intercostobrachial nerve and the medial antebrachial cutaneous nerve that takes its origin from the medial cord of the brachial plexus where it runs superficially, making it extremely difficult to injure both nerves during a midshaft fracture of the humerus.

118 A 63-year-old man had his prostate gland tumor removed 2 years before his present admission to the hospital, complaining of various neurologic problems including headaches. Radiographic examination reveals that the cancer has spread from the pelvis to the posterior cranial fossa by way of the internal vertebral venous plexus (of Batson). During physical examination the patient's right shoulder droops noticeably lower than the left, he exhibits considerable weakness in turning his head to the left, and his tongue points to the right when he attempts to protrude it directly from his mouth. There are no other significant findings. Which of the following nerves are most likely affected? A. Right vagus, right accessory, and right hypoglossal nerves B. Left accessory, right glossopharyngeal, right vagus, and left hypoglossal nerves C. Left hypoglossal, right trigeminal, and left glossopharyngeal nerves D. Right accessory and right hypoglossal nerves E. Left facial, left accessory, right accessory, and vagus nerves

D. Paralysis of the right accessory and hypoglossal nerves is present in this patient. Drooping of the right shoulder occurs as a result of paralysis of the trapezius as a result of injury to the right accessory nerve, which supplies that muscle. Loss of the right accessory nerve would also result in weakness in turning the head to the left, a function of the right sternocleidomastoid muscle, which is supplied by this nerve. The tongue deviation to the right is due to the unopposed activity of the left tongue muscles since the right hypoglossal nerve (which innervates the right tongue muscles) is affected. The other combinations of affected cranial nerves would not produce the specific symptoms described here

24 A 58-year-old male farmer was accidentally struck with a scythe (a long, curved cutting blade) by another worker while they were cutting wheat. He was admitted to the county hospital with severe bleeding. During physical examination the doctor noted that the patient had a foot drop; sensation was present over the dorsum of the foot and the skin of the posterior calf. Which of the following nerves was injured? A. Femoral nerve B. Sciatic nerve C. Superficial fibular (peroneal) nerve D. Deep fibular (peroneal) nerve E. Common fibular (peroneal) nerve

D. The farm instrument has injured the deep fibular (peroneal) branch of the common fibular (peroneal) nerve. It is vulnerable to injury as it arises from the common fibular (peroneal) at the neck of the fibula. The muscles denervated are largely dorsiflexors of the foot; hence, foot drop and a high stepping gait can occur. Sensation on the dorsum of the foot is still present; therefore, the superficial branch is mostly or entirely intact, although sensation between the first and second toes would be absent. Femoral nerve injury would result in loss of knee extension. Loss of the sciatic nerve would result in loss of both the tibial and common fibular (peroneal) nerves. Because plantar flexion is still functional, the tibial nerve has not been cut.

11 A 49-year-old man underwent a coronary bypass graft procedure using the great saphenous vein. Postoperatively, the patient complains of pain and general lack of normal sensation on the medial surface of the leg and foot on the limb from which the graft was harvested. Which nerve was most likely injured during surgery? A. Common fibular (peroneal) B. Superficial fibular (peroneal) C. Lateral sural D. Saphenous E. Tibial

D. The great saphenous vein is commonly used in coronary artery bypass grafts. Because branches of the saphenous nerve cross the vein in the distal part of the leg, the nerve can be damaged if the vein is stripped from the ankle to the knee. Stripping the vein in the opposite direction can protect the nerve and lessen the postoperative discomfort of patients. The saphenous nerve is responsible for cutaneous innervations on the medial surface of the leg and the medial side of the foot. Injury to this nerve will result in a loss of sensation and also can create chronic dysesthesias in the area. The common fibular (peroneal) nerve bifurcates at the neck of the fibula into the superficial and deep fibular (peroneal) nerves, which continue on to innervate the lateral and anterior compartments of the leg, respectively. These nerves are lateral and therefore not associated with the great saphenous vein. The lateral sural nerve is a cutaneous nerve that arises from the junction of branches from the common fibular (peroneal) nerve and tibial nerve and innervates the skin on the posterior aspect of the leg and lateral side of the foot. This nerve is often harvested for nerve grafts elsewhere in the body. The tibial nerve is a terminal branch of the sciatic nerve that continues deep in the posterior compartment of the leg.

159 A 55-year-old right-handed woman presents to the clinic with a 1-week history of right elbow pain. The pain started after a long game of competitive tennis. The pain begins in the elbow and at times radiates into the forearm. Splinting of the elbow decreases the intensity of the pain. During physical examination of the elbow mild swelling and tenderness are noted over the lateral epicondyle. Which one of the following wrist movements, if carried out by the patient with a closed fist and against resistance, will most likely exacerbate the pain? A. Radial deviation B. Ulnar deviation C. Flexion D. Extension E. Flexion and ulnar deviation

D. The lateral epicondyle is the common extensor origin. Most of the extensor muscles of the forearm originate from this area. Putting those muscles in action will exacerbate pain on the lateral epicondyle, a condition nicknamed "tennis elbow." Radial and lateral deviations have no effect because the movement is at the wrist joint. Flexion exacerbates pain on the medial epicondyle if the patient has "golfer's elbow."

73 A 27-year-old woman had suffered a penetrating injury in the popliteal region by an object thrown from a riding lawnmower. She was admitted to the emergency department for removal of the foreign object. After making a midline incision in the skin of the popliteal fossa, the surgical resident observed a vein of moderate size in the superficial tissues. What vein would be expected at this location? A. Popliteal vein B. Perforating tributary to the deep femoral vein C. Great saphenous vein D. Lesser (short) saphenous vein E. Superior medial genicular vein

D. The lesser (short) saphenous vein ascends up the middle of the calf from beneath the lateral malleolus, most commonly terminating at the popliteal fossa by piercing the deep fascia and joining the popliteal vein. The popliteal vein is the most superficial of major structures deep to the deep popliteal fascia. The perforating tributaries of the deep femoral vein drain to the deep femoral vein of the posterior compartment of the thigh, thereafter into the femoral vein. The superior medial genicular vein is a tributary to the popliteal vein

13 A 22-year-old woman is admitted to the emergency department in an unconscious state. The nurse takes a radial pulse to determine the heart rate of the patient. This pulse is felt lateral to which tendon? A. Palmaris longus B. Flexor pollicis longus C. Flexor digitorum profundus D. Flexor carpi radialis E. Flexor digitorum superficialis

D. The location for palpation of the radial pulse is lateral to the tendon of the flexor carpi radialis, where the radial artery can be compressed against the distal radius. The radial pulse can also be felt in the anatomic snuffbox between the tendons of the extensor pollicis brevis and extensor pollicis longus muscles, where the radial artery can be compressed against the scaphoid

177 A 29-year-old woman injures her wrist in a fall on an outstretched hand. Examination reveals pain on movement of the wrist associated with numbness and tingling on the radial side of the palm and palmar aspect of the thumb, index, and middle fingers. A radiograph of the wrist reveals, anterior dislocation of a carpal bone. Which dislocated carpal bone is compressing which structure? A. Pisiform compressing ulnar nerve B. Hook of hamate compressing ulnar artery C. Scaphoid compressing radial artery D. Lunate compressing median nerve E. Trapezoid bone compressing superficial radial nerve

D. The lunate is compressing the median nerve. The pisiform compressing the ulnar nerve is incorrect as the ulnar nerve innervates the skin on the medial one and a half digits. Hook of hamate compressing the ulnar artery is also incorrect. The hook of the hamate forms part of Guyon's canal; compression of the ulnar artery will not produce the deficits described because of the collateral circulation and anastomoses that exist with the radial artery. The scaphoid compressing the radial artery is also incorrect because there is collateral circulation from the palmar arches to compensate for radial artery occlusion. The trapezoid is not compressing the superficial radial nerve because the superficial branch of the radial nerve supplies the radial side and ball of the thumb and radial side of the index finger via its lateral and medial branches

53 A 34-year-old pregnant woman in the maternity ward was experiencing considerable pain during labor. Her obstetrician decided to perform a caudal epidural block. What are the most important bony landmarks used for the administration of such anesthesia? A. Ischial tuberosities B. Ischial spines C. Posterior superior iliac spines D. Sacral cornua E. Coccyx

D. The sacral cornua lie on either side of the sacral hiatus, from which one can gain access to the sacral canal. This is the best landmark for administration of anesthesia. The ischial tuberosities are more commonly used as landmarks for a pudendal nerve block. The ischial spines are only palpated intravaginally. The posterior superior iliac spines, though palpable, are not proximal enough for an epidural block within the sacral canal. The coccyx is not part of the sacral canal.

95 In the radiographs of the knee of a male 28-yearold basketball player, who had apparently suffered a tear in a medial ligament of the knee, the tubercle on the superior aspect of the medial femoral condyle could be seen more clearly than in most individuals. What muscle attaches to this tubercle? A. Semimembranosus B. Gracilis C. Popliteus D. Adductor magnus E. Vastus medialis

D. The tendinous distal portion of the adductor magnus inserts on the adductor tubercle on the upperborder of the medial condyle of the femur. The femoral artery passes through the adductor hiatus proximal to this tendinous band, continuing as the popliteal artery. The semimembranosus inserts on the proximal, posterior portion of the tibia. The gracilis inserts with the pes anserinus on the proximal, medial aspect of the tibia. The popliteus inserts on the distal lateral portion of the femur, just above the origin of the lateral head of gastrocnemius. The vastus medialis inserts with other quadriceps muscle components on the patella and then on to the tibial tuberosity.

48 The swollen and painful left foot of a 23-year-old female long distance runner is examined in the university orthopedic clinic. She states that she stepped on an unseen sharp object while running through the park several days earlier. Emergency surgery is ordered to deal with her tarsal tunnel syndrome. The tarsal tunnel is occupied normally by tendons, vessels, and nerves that pass beneath a very strong band of tissue (flexor retinaculum) on the medial side of the ankle. What is the most anterior of the structures that pass through this tunnel? A. Flexor hallucis longus tendon B. Plantaris tendon C. Tibialis anterior tendon D. Tibialis posterior tendon E. Tibial nerve

D. The tibialis posterior tendon is the most anterior of the structures that pass under the laciniate ligament (flexor retinaculum) on the medial side of the ankle to enter the sole of the foot. Increases of pressure within the tissues of the plantar aspect of the foot, usually due to increased fluid from hemorrhage, inflammatory processes, or infections, cause tarsal tunnel syndrome, comparable to carpal tunnel syndrome of the hand. The plantar aponeurosis and other fibrous and osseous tissues of the plantar surface cause this area to be relatively nondistensible; therefore, it takes little increase of fluid content to result in pressures adequate to restrict venous drainage and, thereafter, arterial inflow to the region. Fasciotomy of the medial skin and fascia of the foot and the posterior compartment of the leg can be required to reduce the pressure and allow healing to take place. The structures that pass beneath the flexor retinaculum are, from anterior to posterior: Tendon of tibialis posterior; tendon of flexor Digitorum longus; posterior tibial Vessels and Nerve; tendon of flexor Hallucis longus. (This is the basis of the mnemonic: "Tom, Dick, and a Very Nervous Harry.") Neither the plantaris tendon nor the tibialis anterior tendon pass through this canal

75 A 56-year-old woman complains of diplopia (double vision) when walking down stairs. A lesion of which of the following nerves is most likely responsible for this patient's complaint? A. Optic B. Oculomotor C. Abducens D. Trochlear E. Frontal

D. The trochlear nerve innervates the superior oblique muscle, which acts to move the pupil downward and laterally. It is the only muscle that can depress the pupil when the eye is adducted. When an individual walks down stairs, this eye motion is initiated, and diplopia results if it is not functioning properly. The optic nerve provides vision, and a lesion of this nerve would not result in diplopia when an affected individual walks down the stairs, but rather diminished vision or blindness. The oculomotor nerve supplies the superior, inferior, and medial rectus as well as the inferior oblique. Overall, innervation from the oculomotor nerve results in upward and inward movements of the eye, and a lesion of this nerve would not induce diplopia in an individual walking down stairs. The abducens nerve innervates the lateral rectus muscle, which abducts the eye, and damage would not induce the diplopia presented in this problem. The frontal nerve is a branch of the ophthalmic division of the trigeminal nerve and provides sensory innervation to the forehead.

129 During a 100-meter sprint a 25-year-old male Olympic athlete suddenly pulls up in discomfort and is seen to be clutching the back of his left thigh in agony. Upon further examination the athlete describes the pain as a "tearing" sensation and is unable to flex his knee. Based on these symptoms which of the following actions are affected due to this injury? A. Flexion of the hip and extension of the knee B. Extension of the hip and dorsiflexion C. Medial rotation of the hip D. Lateral rotation of the hip E. Hip extension and knee flexion

E. Because the hamstrings cross two joints and are very crucial during all phases of running, but especially during the late swing through midstance phase of running, are easily injured. Their normal action includes hip extension and knee flexion. The do not rotate the hip

16 A 15-year-old woman was suspected of having meningitis. To obtain a sample of cerebrospinal fluid by spinal tap in the lumbar region (lumbar puncture), the tip of the needle must be placed in which of the following locations? A. In the epidural space B. Between anterior and posterior longitudinal ligaments C. Superficial to the ligamentum flavum D. Between arachnoid mater and dura mater E. In the subarachnoid space

E. CSF is found within the subarachnoid space and is continuous with the ventricles of the brain (CSF flows from the ventricles to the subarachnoid space). The epidural space, positioned between the dura mater and periosteum, contains fat and the internal vertebral venous plexus (of Batson). The subdural space, between the arachnoid mater and dura mater, exists only as a potential space and does not contain cerebrospinal fluid. The anterior and posterior longitudinal ligaments traverse the length of the vertebral bodies

84 A 14-year-old girl accidentally flipped her bicycle off a curb, fell, and landed on her face. Although she was wearing a helmet, she landed in such a way that her neck was forced into hyperextension. Which of the following ligaments of the cervical spine was stretched to the greatest degree during her injury? A. Posterior longitudinal ligament B. Ligamentum nuchae C. Ligamenta flava D. Supraspinous ligament E. Anterior longitudinal ligament

E. Ligaments serve to restrict movement. The anterior longitudinal ligament courses downward on the anterior surface of the vertebral bodies attaching to the intervertebral discs along its way. It is stretches from the base of the skull inferiorly to the anterior surface of the sacrum. The anterior longitudinal ligament is the most anteriorly positioned ligament of the vertebral column and limits its extension. The posterior longitudinal ligament travels on the posterior surface of the vertebral bodies attaching to the intervertebral discs along the way. This ligament serves to prevent excessive flexion of the vertebral column and extends from C2 to the sacrum. The supraspinous ligament attaches the tips of the spinous processes to each other from C7 to the sacrum. Superiorly the ligament broadens becoming more distinct and triangular and is termed the ligamentum nuchae. Ligamentum nuchae limits excessive flexion of the cervical spine and serves as an attachment for muscles. Ligamentum flava attach the internal surfaces of adjacent lamina to each other and prevent them from pulling apart during flexion.

142 After being struck from behind by a motor vehicle, a 55-year-old man presents to the hospital with a swelling of his right knee. Imaging reveals a large hematoma of the popliteal artery compressing his tibial nerve. Upon neurologic examination which movement would likely be diminished in strength? A. Dorsiflexion of the foot B. Flexion of the thigh C. Extension of the digits D. Extension of the leg E. Plantar flexion of the foot

E. Plantar flexion is mostly due to the gastrocnemius and soleus muscles, which are supplied by the tibial nerve. The tibial nerve leaves the popliteal fossa by passing deep to the gastrocnemius and soleus muscles and lies posterior to the popliteal artery. Therefore a hematoma of the popliteal artery will also compress the nerve. Dorsiflexion of the foot is due to contraction of the muscles in the anterior compartment of the leg

53 A 54-year-old male cotton farmer visits the outpatient clinic because of a penetrating injury to his forearm from a baling hook. After the limb is anesthetized, the site of the wound is opened and flushed thoroughly to remove all debris. The patient is not able to oppose the tip of the thumb to the tip of the index finger, as in making the OK sign. He is able to touch the tips of the ring and little fingers to the pad of his thumb. What nerve has most likely been injured? A. Median B. Posterior interosseous C. Radial D. Recurrent median E. Anterior interosseous

E. The anterior interosseous nerve is a branch of the median nerve that supplies the flexor pollicis longus, the lateral half of the flexor digitorum profundus, and the pronator quadratus muscles. If it is injured, flexion of the interphalangeal joint of the thumb will be compromised. The median nerve gives rise to the anterior interosseous nerve but is not a direct enough answer as injury to it would result in more widespread effects. The posterior interosseous nerve supplies extensors in the forearm, not flexors. The radial nerve gives rise to the posterior interosseous nerve and is not associated with the anterior interosseous nerve; therefore, it would not have any effect on the flexors of the forearm. The recurrent median nerve is also a branch of the median nerve but supplies the thenar eminence muscles, and its injury would result in problems with opposable motion of the thumb

27 Following several days of 12-hour daily rehearsals of the symphony orchestra for a performance of a Wagnerian opera, the 52-year-old male conductor experienced such excruciating pain in the posterior aspect of his right forearm that he could no longer direct the musicians. When the maestro's forearm was palpated 2 cm distal and posteromedial to the lateral epicondyle, the resulting excruciating pain caused him to grimace. Injections of steroids and rest were recommended to ease the pain. Which of the following injuries is most likely? A. Compression of the ulnar nerve by the flexor carpi ulnaris B. Compression of the median nerve by the pronator teres C. Compression of the median nerve by the flexor digitorum superficialis D. Compression of the superficial radial nerve by the brachioradialis E. Compression of the deep radial nerve by the supinator

E. The deep branch of the radial nerve courses between the two heads of the supinator muscle and is located just medial and distal to the lateral epicondyle. After the nerve emerges from the supinator it is called the posterior interosseous nerve. It can be irritated by hypertrophy of the supinator, which compresses the nerve, causing pain and weakness. The ulnar nerve courses laterally behind the medial epicondyle and continues anterior to the flexor carpi ulnaris muscle. The median nerve passes into the forearm flexor compartment; the superficial radial nerve courses down the lateral aspect of the posterior forearm and would not cause pain due to pressure applied to the posterior forearm

131 A 22-year-old woman is admitted to the hospital after falling from a tree. Radiographic examination reveals fractured pisiform and hamate bones. Which of the following nerves will most likely be injured? A. Median B. Recurrent median C. Radial D. Anterior interosseous E. Deep ulnar

E. The deep branch of the ulnar nerve arises at the level of the pisiform bone and passes between the pisiform and the hook of the hamate; hence the deep branch of the ulnar nerve is most likely to be injured in this patient. The median nerve enters the forearm between the humeral and ulnar heads of the pronator teres muscle then becomes superficial near the wrist. The recurrent branch of the median nerve branches off after the median nerve enters the palm through the carpal tunnel. The radial nerve divides into superficial and deep branches when it enters the cubital fossa

153 A 45-year-old man is admitted to the emergency department after a fall and subsequent leg injury. On physical examination the patient has a foot drop but eversion is unaffected. Which nerve is most likely injured? A. Tibial B. Common fibular (peroneal) C. Superficial fibular (peroneal) D. Saphenous E. Deep fibular (peroneal)

E. The deep fibular (peroneal) nerve along with the superficial fibular (peroneal) nerve are branches of the common fibular nerve. The deep fibular (peroneal) nerve innervates muscles of the anterior compartment of the leg dorsiflexors of the foot and the skin between the great toe and second toes, while the superficial fibular nerve innervates the lateral compartment muscles of the leg, which are evertors of the foot and the skin on most of the dorsum of the foot. If the common fibular nerve were damaged all the structures that receive innervation via this nerve will be compromising dorsiflexion and eversion. Damage to the superficial fibular nerve affects the ability to evert the foot but does not result in foot drop, making the deep fibular nerve, which innervates the dorsiflexors of the foot, the best choice as the injury describes foot drop with the ability to evert the foot conserved. The saphenous nerve is a cutaneous nerve, while the tibial nerve innervates posterior compartment muscles.

4 A 45-year-old man is treated at the hospital after he fell from his bicycle. Radiologic examination reveals fractures both of the tibia and the fibula. On physical examination the patient has a foot drop, but normal eversion (Fig. 5-1). Which of the following nerves is most likely injured? A. Tibial B. Common fibular (peroneal) C. Superficial fibular (peroneal) D. Saphenous E. Deep fibular (peroneal)

E. The deep fibular (peroneal) nerve is responsible for innervating the muscles of the anterior compartment of the leg, which are responsible for toe extension, foot dorsiflexion, and inversion. Injury to this nerve will result in foot drop and also loss of sensation between the first and second toes. Injury to the tibial nerve affects the posterior compartment muscles of the leg, which are responsible for plantar flexion and toe flexion, as well as the intrinsic muscles of the sole of the foot. The common fibular (peroneal) nerve splits into the superficial and deep fibular (peroneal) nerves, and these supply both the lateral and anterior compartments. The superficial fibular (peroneal) nerve innervates the fibularis (peroneus) longus and brevis muscles, which provide eversion of the foot. If the common fibular (peroneal) nerve were injured, eversion of the foot and plantar flexion would be lost in addition to dorsiflexion and inversion. The saphenous nerve, a continuation of the femoral nerve, is a cutaneous nerve that supplies the medial side of the leg and foot and provides no motor innervation.

100 A 38-year-old woman has been in labor for 14 hours and has agreed to have an epidural anesthetic injection for pain control. Which of the following structures is most likely to be the last penetrated by the needle before it reaches the epidural space? A. Supraspinous ligament B. Interspinous ligament C. Anterior longitudinal ligament D. Posterior longitudinal ligament E. Ligamenta flava

E. The ligamentum flavum lies within the vertebral canal on the anterior aspect of the vertebral arches connecting the lamina of adjacent vertebrae. Puncturing this ligament allows the needle to enter into the epidural/extradural space for the injection of the anesthetic. Although the posterior longitudinal ligament lies within the spinal canal, it will not be punctured during the procedure. The supraspinous ligament connects and passes along the tips of the vertebral spinous processes. The interspinous ligament lies between adjacent spinous processes. The anterior longitudinal ligament connects the anterior aspect of the vertebral body. These ligaments do not lie within the vertebral canal

33 As she fell from the uneven parallel bars, a 17-year-old female gymnast grasped the lower bar briefly with one hand but then fell painfully to the floor. An MRI examination reveals an injury to the medial cord of the brachial plexus. Which of the following spinal nerve levels would most likely be affected? A. C5, C6 B. C6, C7 C. C7, C8 D. C7, C8, T1 E. C8, T1

E. The medial cord has been injured by traction on the lower trunk of the brachial plexus. The medial cord is the continuation of the inferior (lower) trunk of the brachial plexus, which is formed by C8 and T1. C5 and C6 are typically associated with the superior (upper) trunk level and thus the lateral cord. C7 forms the middle trunk. An injury to the posterior cord would usually involve the C7 spinal nerve. This is a typical Klumpke paralysis

34 Lower limb angiography of an 82-year-old woman reveals a possible cause for her limb pain during her workout routines in the health spa. The artery that was occluded is one that should have been demonstrable passing between the proximal part of the space between the tibia and fibula. Which of the following arteries is most likely affected? A. Deep femoral B. Popliteal C. Posterior tibial D. Fibular (peroneal) E. Anterior tibial

E. The popliteal artery is the continuation of the femoral artery after it passes through the hiatus of the adductor magnus. The popliteal artery divides into the anterior and posterior tibial arteries. The anterior tibial artery passes between the tibia and fibula proximally in the posterior compartment of the leg, whereas the posterior tibial artery continues in the posterior compartment of the leg, to its division into medial and lateral plantar arteries. The posterior tibial artery provides origin for the fibular (peroneal) artery, which supplies the lateral compartment of the leg. The deep femoral artery provides origin for the three or four perforating branches that supply the posterior compartment of the thigh.

52 A 17-year-old boy suffered the most common of fractures of the carpal bones when he fell on his outstretched hand. Which bone would this be? A. Trapezium B. Lunate C. Pisiform D. Hamate E. Scaphoid

E. The scaphoid bone is the most commonly fractured carpal bone.

45 A 45-year-old woman is admitted to the hospital for severe ear pain. Physical examination reveals chronic infection of the mastoid air cells (mastoiditis). The infection can erode the thin layer of the bone between the mastoid air cells and the posterior cranial fossa and spread most commonly into which of the following venous structures? A. Superior sagittal sinus B. Inferior sagittal sinus C. Straight sinus D. Cavernous sinus E. Sigmoid sinus

E. The sigmoid venous sinus empties into the internal jugular vein and drains the cranial vault. It runs along the posterior cranial fossa near the suture between the temporal and occipital bones just lateral to the mastoid air cells. The superior sagittal sinus lies within the superior aspect of the longitudinal fissure, between the two cerebral hemispheres. The inferior sagittal sinus runs inferior to the superior sagittal sinus within the falx cerebri and joins the great cerebral vein (of Galen) to form the straight sinus. The straight sinus drains the great cerebral vein (of Galen) into the confluence of sinuses. The cavernous sinus is located within the middle cranial fossa and receives the ophthalmic veins, the greater petrosal sinus, and other venous vessels

76 A 43-year-old man is admitted to the hospital complaining of diplopia (double vision) when walking down stairs. During physical examination of the extraocular muscles the patient experiences diplopia, and when he is asked to turn his right eye inward toward his nose and look down, he is able to look inward but not down. Which nerve is most likely involved? A. Abducens B. Nasociliary C. Oculomotor, inferior division D. Oculomotor, superior division E. Trochlear

E. The superior oblique muscle turns the pupil downward from the adducted position. Inability to perform this motion, in conjunction with diplopia when walking down stairs, indicates damage to the trochlear nerve. The abducens innervates the lateral rectus, resulting in abduction of the eye. The oculomotor nerve supplies the superior, inferior, and medial rectus as well as the inferior oblique muscles. Overall, innervation from the oculomotor nerve results in upward and downward movements of the eye. Damage to this nerve would not induce diplopia when an affected individual walks down stairs. In addition, inability to gaze downward in the adducted position does not indicate oculomotor nerve damage. In this position the oculomotor nerve would be responsible for upward movement. The nasociliary nerve is a sensory nerve originating from the ophthalmic branch of the trigeminal nerve.

37 A 20-year-old male hiker suffers a deep puncture wound during a fall. Physical examination reveals a lesion between the trapezius and latissimus dorsi muscles on the right lateral side of his back. Upon admission to the hospital, physical examination reveals weak adduction and medial rotation of his arm. Which of the following muscles is most probably injured? A. Teres minor B. Triceps brachii C. Supraspinatus D. Infraspinatus E. Teres major

E. The teres major is responsible for adduction and medial rotation of the humerus, the teres minor is responsible for lateral rotation of the humerus, the triceps brachii is responsible for extension of the forearm, the supraspinatus is responsible for the first 0 to 15 degrees of abduction, and the infraspinatus is a lateral rotator.


Kaugnay na mga set ng pag-aaral

A Whole Bunch of Sentence Patterns

View Set

Chapter 18 EAQ: Eating & Feeding Disorders

View Set

MKTG - Chapter 14 Practice Questions

View Set

Concepts of Programming Languages - Chapter 5 (Names, Bindings, and Scopes) Part 2

View Set